Difference between revisions of "1989 AIME Problems/Problem 11"

m (Solution: ->tex)
m (wikify)
Line 1: Line 1:
 
== Problem ==
 
== Problem ==
A sample of 121 integers is given, each between 1 and 1000 inclusive, with repetitions allowed. The sample has a unique mode (most frequent value). Let <math>D^{}_{}</math> be the difference between the mode and the arithmetic mean of the sample. What is the largest possible value of <math>\lfloor D^{}_{}\rfloor</math>? (For real <math>x^{}_{}</math>, <math>\lfloor x^{}_{}\rfloor</math> is the greatest integer less than or equal to <math>x^{}_{}</math>.)
+
A sample of 121 [[integer]]s is given, each between 1 and 1000 inclusive, with repetitions allowed. The sample has a unique [[mode]] (most frequent value). Let <math>D^{}_{}</math> be the difference between the mode and the [[arithmetic mean]] of the sample. What is the largest possible value of <math>\lfloor D^{}_{}\rfloor</math>? (For real <math>x^{}_{}</math>, <math>\lfloor x^{}_{}\rfloor</math> is the [[floor function|greatest integer]] less than or equal to <math>x^{}_{}</math>.)
  
 
== Solution ==
 
== Solution ==
Line 6: Line 6:
  
 
== See also ==
 
== See also ==
* [[1989 AIME Problems/Problem 12|Next Problem]]
+
{{AIME box|year=1989|num-b=10|num-a=12}}
* [[1989 AIME Problems/Problem 10|Previous Problem]]
 
* [[1989 AIME Problems]]
 

Revision as of 18:48, 7 March 2007

Problem

A sample of 121 integers is given, each between 1 and 1000 inclusive, with repetitions allowed. The sample has a unique mode (most frequent value). Let $D^{}_{}$ be the difference between the mode and the arithmetic mean of the sample. What is the largest possible value of $\lfloor D^{}_{}\rfloor$? (For real $x^{}_{}$, $\lfloor x^{}_{}\rfloor$ is the greatest integer less than or equal to $x^{}_{}$.)

Solution

It is obvious that there will be $n+1$ values equal to one and $n$ values each of $1000, 999, 998 \ldots$. It is fairly easy to find the maximum. Try $n=1$, which yields $924$, $n=2$, which yields $942$, $n=3$, which yields $947$, and $n=4$, which yields $944$. The maximum difference occurred at $n=3$, so the answer is $947$.

See also

1989 AIME (ProblemsAnswer KeyResources)
Preceded by
Problem 10
Followed by
Problem 12
1 2 3 4 5 6 7 8 9 10 11 12 13 14 15
All AIME Problems and Solutions